LSAT and Law School Admissions Forum

Get expert LSAT preparation and law school admissions advice from PowerScore Test Preparation.

User avatar
 sunshine123
  • Posts: 44
  • Joined: Jul 18, 2022
|
#98665
Hello,

I have a general question about multiple-speaker problems like this one that I would greatly appreciate having answered. When the second speaker says something bare like, "I disagree." as in this case, does that mean/ can we infer, based on that statement alone, that they disagree with the conclusion of the first speaker?

Best,
Sunshine
 Rachael Wilkenfeld
PowerScore Staff
  • PowerScore Staff
  • Posts: 1379
  • Joined: Dec 15, 2011
|
#98733
Generally, yes, Sunshine. However, be careful that there's no other context for the disagreement statement. It's possible that there could be further information in the statement that would contextualize the disagreement as a disagreement with a fact or assumption of the first speaker.

Good question!
User avatar
 AnaSol
  • Posts: 19
  • Joined: Nov 20, 2023
|
#106233
Hi,

I'm very confused by this question.

Where is the flaw precisely? That Arjun said 'could'? If Arjun had responded something like: "I disagree! For example, a local hospital had an unauthorized use of medical records systems , which damaged data systems on which human lives depended, and therefore computer crimes also cause physical harm to people.

Or something to that effect, where something did happen vs that it could have happened, would that eliminate the flaw?

Thanks!
User avatar
 Jeff Wren
PowerScore Staff
  • PowerScore Staff
  • Posts: 419
  • Joined: Oct 19, 2022
|
#106254
Hi AnaSol,

That's exactly right!

In Arjun's premise, all that is stated is that medical records in hospitals could be damaged, which could harm human lives. Arjun's conclusion shifts from "could" to "does." By saying "computer crimes also cause physical harm," Arjun is saying that this has in fact occurred, not just that it is theoretically possible.

The shift in language between premises and conclusion is a common problem in LR arguments. Sometimes it's a shift in certainty (what's possible, what's likely, what's certain), other times it may be quantity (some, most, all), other times it shifts in tense (past, present, future).

You're example is almost correct, but it would not only need to show that there was a data breach at a hospital, but that a patient was actually harmed because of the breach (such as getting the wrong medication that harmed the patient due to the data breach). If this kind of incident had been shown, then the conclusion would be properly supported.

Get the most out of your LSAT Prep Plus subscription.

Analyze and track your performance with our Testing and Analytics Package.